Đến nội dung

Hình ảnh

Topic về Bất đẳng thức, cực trị THCS


  • Please log in to reply
Chủ đề này có 1205 trả lời

#1121
kienvuhoang

kienvuhoang

    Thượng sĩ

  • Thành viên
  • 202 Bài viết

Cho x,y,z>0 thỏa mãn x+y+z=1. Tìm giá trị lớn nhất của biểu thức: P=$\frac{x}{x+1}+\frac{y}{y+1}+\frac{z}{z+1}$

Ta có:$\frac{1}{x+1}+\frac{1}{y+1}+\frac{1}{z+1}\geq \frac{9}{x+y+z+3}=\frac{9}{4}$

$\Rightarrow P\leq \frac{3}{4}$



#1122
NHoang1608

NHoang1608

    Sĩ quan

  • Thành viên
  • 375 Bài viết

Ta có:$\frac{1}{x+1}+\frac{1}{y+1}+\frac{1}{z+1}\geq \frac{9}{x+y+z+3}=\frac{9}{4}$

$\Rightarrow P\leq \frac{3}{4}$

$\sum{\frac{x}{x+1}}=\sum{\frac{x}{x+y+x+z}} \leq \frac{1}{4}\sum{(\frac{x}{x+y}+\frac{x}{x+z})}=\frac{3}{4}$


Bài viết đã được chỉnh sửa nội dung bởi NHoang1608: 14-03-2017 - 22:32

The greatest danger for most of us is not that our aim is too high and we miss it, but that it is too low and we reach it.

----- Michelangelo----


#1123
kienvuhoang

kienvuhoang

    Thượng sĩ

  • Thành viên
  • 202 Bài viết

$\sum{\frac{x}{x+1}}=\sum{\frac{x}{x+y+x+z}} \leq \frac{1}{4}\sum{(\frac{x}{x+y}+\frac{x}{x+z})}=\frac{3}{4}$

Cách này có vẻ dài hơn cách của mình



#1124
05479865132

05479865132

    Binh nhì

  • Thành viên mới
  • 19 Bài viết

Cho x,y,z>0.CMR $(1+\frac{x}{y})(1+\frac{y}{z})(1+\frac{z}{x})\geq 2+\frac{2(x+y+z)}{\sqrt[3]{xyz}}$



#1125
Nguyenphuctang

Nguyenphuctang

    Sĩ quan

  • Banned
  • 499 Bài viết

Cho x,y,z>0.CMR $(1+\frac{x}{y})(1+\frac{y}{z})(1+\frac{z}{x})\geq 2+\frac{2(x+y+z)}{\sqrt[3]{xyz}}$

Đây là 1 bài quen thuộc: APMO 1998.

$\Leftrightarrow \sum \frac{x}{y}\geq \frac{x+y+z}{\sqrt[3]{xyz}}$

Theo bất đẳng thức AM GM:

$3\sum \frac{x}{y}=\sum \left ( \frac{2x}{y}+\frac{y}{z} \right )\geq 3\sum \frac{x}{\sqrt[3]{xyz}}\Rightarrow \sum \frac{x}{y}\geq \frac{x y+z}{\sqrt[3]{xyz}}$

Suy ra đpcm



#1126
Maths2017

Maths2017

    Lính mới

  • Thành viên mới
  • 5 Bài viết

Cho a,b,c,d $\epsilon \mathbb{R}$ thỏa mãn $a^{2}+b^{2}+c^{2}+d^{2}=1$ .
GTLN ab+ac+ca+3ad


Bài viết đã được chỉnh sửa nội dung bởi Maths2017: 30-03-2017 - 20:08


#1127
dat102

dat102

    Trung sĩ

  • Thành viên
  • 150 Bài viết

cho các số thực $a,b,c$. Chứng minh :

$$\frac{ab}{a+b-c}+\frac{bc}{b+c-a}+\frac{ca}{c+a-b}\ge a+b+c$$


:ukliam2:  $\sqrt{MF}$  :ukliam2: 


#1128
NHoang1608

NHoang1608

    Sĩ quan

  • Thành viên
  • 375 Bài viết

Cho 3 số dương x,y,z thõa mãn x+y+z=$\frac{3}{4}$ .Chứng minh rằng

$6(x^{2}+y^{2}+z^{2})+10(xy+yz+xz)+2(\frac{1}{2x+y+z}+\frac{1}{x+2y+z}+\frac{1}{x+y+2z})\geq 9$

Áp dụng bđt $Cauchy-Schwarzt$ thì ta được: $\frac{1}{2x+y+z}+\frac{1}{2y+x+z}+\frac{1}{2z+x+y} \geq \frac{9}{4(x+y+z)}=3$

                                                                        $\Rightarrow 2(\frac{1}{2x+y+z}+\frac{1}{2y+x+z}+\frac{1}{2z+x+y})\geq 6$ $(1)$

Lại có $6(x^{2}+y^{2}+z^{2})+10(xy+yz+zx)=5(x+y+z)^{2}+x^{2}+y^{2}+z^{2} \geq 5(x+y+z)^{2}+\frac{(x+y+z)^{2}}{3}= 3$ $(2)$

Cộng $(2)$ với $(1)$ thì được điều $ĐPCM$.


Bài viết đã được chỉnh sửa nội dung bởi NHoang1608: 30-03-2017 - 20:07

The greatest danger for most of us is not that our aim is too high and we miss it, but that it is too low and we reach it.

----- Michelangelo----


#1129
NHoang1608

NHoang1608

    Sĩ quan

  • Thành viên
  • 375 Bài viết

cho các số thực $a,b,c$. Chứng minh :

$$\frac{ab}{a+b-c}+\frac{bc}{b+c-a}+\frac{ca}{c+a-b}\ge a+b+c$$

Đặt $(a+b-c;b+c-a;c+a-b)=(x;y;z) \Rightarrow \frac{x+y}{2}=b; \frac{y+z}{2}=c ;\frac{x+z}{2}=a$

Suy ra $BĐT \Leftrightarrow \frac{(x+y)(x+z)}{4x}+\frac{(y+z)(y+x)}{4y}+\frac{(z+x)(z+y)}{4z} \geq x+y+z$

                    $\Leftrightarrow (x+y)(y+z)(z+x)[\frac{1}{4x(y+z)}+\frac{1}{4y(x+z)}+\frac{1}{4z(x+y)}] \geq x+y+z$ $(*)$

Áp dụng bđt thân quen ta có: $(x+y)(y+z)(z+x) \geq \frac{8}{9}(x+y+z)(xy+yz+zx)$ $(1)$

Ap dụng bđt $Cauchy-Schwazt$ thì ta được $\frac{1}{4x(y+z)}+\frac{1}{4y(x+z)}+\frac{1}{4z(x+y)} \geq \frac{9}{8(xy+yz+zx)}$ $(2)$

Nhân $(1)$ với $(2)$ thì ta được $(x+y)(y+z)(z+x)[\frac{1}{4x(y+z)}+\frac{1}{4y(x+z)}+\frac{1}{4z(x+y)}]\geq  \frac{9}{8(xy+yz+zx)}.\frac{8}{9}(x+y+z)(xy+yz+zx)=x+y+z$ suy ra $(*)$ đúng $\Rightarrow ĐPCM$.

Đẳng thức xảy ra khi $a=b=c$.


Bài viết đã được chỉnh sửa nội dung bởi NHoang1608: 30-03-2017 - 20:27

The greatest danger for most of us is not that our aim is too high and we miss it, but that it is too low and we reach it.

----- Michelangelo----


#1130
05479865132

05479865132

    Binh nhì

  • Thành viên mới
  • 19 Bài viết

Cho a,b,c là các số thực dương thỏa mãn abc=1.Chứng minh:

$\frac{a}{(a+1)(b+1)}+\frac{b}{(b+1)(c+1)}+\frac{c}{(c+1)(a+1)}\geq \frac{3}{4}$

Cho ba số dương a,b,c thỏa $a+b+c\leq k$ thì $(1+\frac{1}{a})(1+\frac{1}{b})(1+\frac{1}{c})\geq (1+\frac{3}{k})^3$



#1131
viet9a14124869

viet9a14124869

    Trung úy

  • Thành viên
  • 903 Bài viết

Cho a,b,c là các số thực dương thỏa mãn abc=1.Chứng minh:

$\frac{a}{(a+1)(b+1)}+\frac{b}{(b+1)(c+1)}+\frac{c}{(c+1)(a+1)}\geq \frac{3}{4}$

Cho ba số dương a,b,c thỏa $a+b+c\leq k$ thì $(1+\frac{1}{a})(1+\frac{1}{b})(1+\frac{1}{c})\geq (1+\frac{3}{k})^3$

Câu đầu đồng quy biểu thức ,,,ta cần chứng minh $ab+bc+ca+a+b+c\geq 6$ đúng theo AM-GM

Câu sau ta dùng holder với AM-GM ta có $\prod (\frac{1}{2}+\frac{1}{2}+\frac{1}{a})\geq (1+\frac{1}{\sqrt[3]{abc}})^3\geq (1+\frac{3}{a+b+c})^3\geq (1+\frac{3}{k})^3$


                                                                    SÓNG BẮT ĐẦU TỪ GIÓ

                                                                    GIÓ BẮT ĐẦU TỪ ĐÂU ?

                                                                    ANH CŨNG KHÔNG BIẾT NỮA 

                                                                    KHI NÀO...? TA YÊU NHAU .


#1132
The Flash

The Flash

    Trung sĩ

  • Thành viên
  • 190 Bài viết

Cho $a,b,c$ là các số thực dương thỏa mãn $a+b+c=1$

Tìm GTLN của $A=a(b^2+c^2)+b(c^2+a^2)+c(a^2+b^2)$


Bài viết đã được chỉnh sửa nội dung bởi The Flash: 04-04-2017 - 11:23


#1133
NTMFlashNo1

NTMFlashNo1

    Sĩ quan

  • Thành viên
  • 344 Bài viết

Cho $a,b,c$ là các số thực dương thỏa mãn $a+b+c=1$

Tìm GTLN của $A=a(b^2+c^2)+b(c^2+a^2)+c(a^2+b^2)$

Đặt $a+b+c=p;ab+bc+ca=q;abc=r$

Ta có:

$A=\sum a(a^{2}+b^{2})=\sum ab(a+b)=pq-3r\leq pq-\frac{p(4q-p^{2})}{9}=\frac{5pq+p^{3}}{9}\leq \frac{\frac{5}{3}p^{3}+p^{3}}{9}=\frac{8}{27}p^{3}=\frac{8}{27}$

Dấu $'='$ xảy ra khi và chỉ khi $a=b=c=\frac{1}{3}$

 

 

 

P/s: Tham khảo cách đặt $p,q,r$ tại đây

 

 

Sorry bác; làm nhầm rồi

File gửi kèm


Bài viết đã được chỉnh sửa nội dung bởi NTMFlashNo1: 05-04-2017 - 11:36

$\boxed{\text{Nguyễn Trực-TT-Kim Bài secondary school}}$


#1134
05479865132

05479865132

    Binh nhì

  • Thành viên mới
  • 19 Bài viết

Cho a,b,c>0 và a^2+b^2+c^2=1.CM $\frac{bc}{a^2+1}+\frac{ca}{b^2+1}+\frac{ab}{c^2+1}\leqslant \frac{3}{4}$


Bài viết đã được chỉnh sửa nội dung bởi 05479865132: 08-04-2017 - 21:09


#1135
viet9a14124869

viet9a14124869

    Trung úy

  • Thành viên
  • 903 Bài viết

Cho a,b,c>0 và a^2+b^2+c^2=1.CM $\frac{bc}{a^2+1}+\frac{ca}{b^2+1}+\frac{ab}{c^2+1}\leqslant \frac{3}{4}$

Ta có $\frac{b^2}{a^2+b^2}+\frac{c^2}{c^2+a^2}\geq \frac{(b+c)^2}{2a^2+b^2+c^2}\geq \frac{4bc}{2a^2+b^2+c^2}=\frac{4bc}{a^2+1}$

Làm tương tự rồi cộng 3 vế lại ta có đpcm !

Dấu = xảy ra khi $a=b=c=\frac{1}{\sqrt{3}}$


                                                                    SÓNG BẮT ĐẦU TỪ GIÓ

                                                                    GIÓ BẮT ĐẦU TỪ ĐÂU ?

                                                                    ANH CŨNG KHÔNG BIẾT NỮA 

                                                                    KHI NÀO...? TA YÊU NHAU .


#1136
AnhTran2911

AnhTran2911

    Thượng sĩ

  • Thành viên
  • 230 Bài viết

 

 

Đặt $(a+b-c;b+c-a;c+a-b)=(x;y;z) \Rightarrow \frac{x+y}{2}=b; \frac{y+z}{2}=c ;\frac{x+z}{2}=a$

Suy ra $BĐT \Leftrightarrow \frac{(x+y)(x+z)}{4x}+\frac{(y+z)(y+x)}{4y}+\frac{(z+x)(z+y)}{4z} \geq x+y+z$

                    $\Leftrightarrow (x+y)(y+z)(z+x)[\frac{1}{4x(y+z)}+\frac{1}{4y(x+z)}+\frac{1}{4z(x+y)}] \geq x+y+z$ $(*)$

Áp dụng bđt thân quen ta có: $(x+y)(y+z)(z+x) \geq \frac{8}{9}(x+y+z)(xy+yz+zx)$ $(1)$

Ap dụng bđt $Cauchy-Schwazt$ thì ta được $\frac{1}{4x(y+z)}+\frac{1}{4y(x+z)}+\frac{1}{4z(x+y)} \geq \frac{9}{8(xy+yz+zx)}$ $(2)$

Nhân $(1)$ với $(2)$ thì ta được $(x+y)(y+z)(z+x)[\frac{1}{4x(y+z)}+\frac{1}{4y(x+z)}+\frac{1}{4z(x+y)}]\geq  \frac{9}{8(xy+yz+zx)}.\frac{8}{9}(x+y+z)(xy+yz+zx)=x+y+z$ suy ra $(*)$ đúng $\Rightarrow ĐPCM$.

Đẳng thức xảy ra khi $a=b=c$.

liệu a, b, c thực thì có đúng k bạn


        AQ02

                                 


#1137
TrBaoChis

TrBaoChis

    Hạ sĩ

  • Banned
  • 81 Bài viết

cho các số thực $a,b,c$. Chứng minh :

$$\frac{ab}{a+b-c}+\frac{bc}{b+c-a}+\frac{ca}{c+a-b}\ge a+b+c$$

với (a,b,c)= (-2,-3,4) thì BĐT sai 



#1138
NHoang1608

NHoang1608

    Sĩ quan

  • Thành viên
  • 375 Bài viết
Đề ra cho là sai $a,b,c\ge 0$ mới đúng

The greatest danger for most of us is not that our aim is too high and we miss it, but that it is too low and we reach it.

----- Michelangelo----


#1139
viet9a14124869

viet9a14124869

    Trung úy

  • Thành viên
  • 903 Bài viết

cho các số thực $a,b,c$. Chứng minh :

$$\frac{ab}{a+b-c}+\frac{bc}{b+c-a}+\frac{ca}{c+a-b}\ge a+b+c$$

 

Đề ra cho là sai $a,b,c\ge 0$ mới đúng

Mình nghĩ nên có thêm điều kiện a,b,c là 3 cạnh tam giác sẽ đúng hơn ,,bởi nếu a=1,b=2,c=4 thì bất đẳng thức sai


                                                                    SÓNG BẮT ĐẦU TỪ GIÓ

                                                                    GIÓ BẮT ĐẦU TỪ ĐÂU ?

                                                                    ANH CŨNG KHÔNG BIẾT NỮA 

                                                                    KHI NÀO...? TA YÊU NHAU .


#1140
haccau

haccau

    Binh nhất

  • Thành viên
  • 47 Bài viết

1. Cho 3 số dương a,b,c. CMR: $(\frac{a}{b}+\frac{b}{c}+\frac{c}{a})^{2}\geq(a+b+c)(\frac{1}{a}+\frac{1}{b}+\frac{1}{c})$

2. Cho các só thực x,y, z thỏa mãn điều kiện: x2 +y+z2=1. Tìm GTLN của A=xy+yz+2xz

3. Cho các số thực a,b,c thuộc đoạn [-2;5] tm: a+2b+3c$\leq$2. Tìm GTLN: a2+2b2+3c2

4. Cho a,b,c>0 tm: $\frac{1}{a}+\frac{1}{b}+\frac{1}{c}$=1. CMR:$\frac{a^{2}}{a+bc}+\frac{b^{2}}{b+ca}+\frac{c^{2}}{c+ab}\geq \frac{a+b+c}{4}$


:lol:  :lol:  :lol: Don't let your dreams just be dreams!!! :lol:  :lol:  :lol: 





0 người đang xem chủ đề

0 thành viên, 0 khách, 0 thành viên ẩn danh